A bag contains a blue ball, some red balls, and some green balls.
You reach into the bag and pull out three balls at random.
The probability you pull out one of each color is exactly 3%.
How many balls were initially in the bag?
This section requires Javascript.
You are seeing this because something didn't load right. We suggest you, (a) try
refreshing the page, (b) enabling javascript if it is disabled on your browser and,
finally, (c)
loading the
non-javascript version of this page
. We're sorry about the hassle.
Nice question. I guess the trickiest part is to find a 'clean' way of solving the equation
2 0 0 R G = ( R + G ) 3 − ( R + G ) for positive integers R , G . (I resorted to WolframAlpha.)
Log in to reply
First show that by rational root theorem that if R = G is a solution, then the cubic equation yields no solution, so R = G .
WLOG R > G > 0 , then by AM-GM inequality , we have R G < 2 R + G ⇔ R G < 4 ( R + G ) 2 ⇔ 2 0 0 R G < 5 0 ( R + G ) 2 .
Let Y = R + G ≥ 1 + 1 = 2 , then we want to find the integer solution for 5 0 Y 2 > Y 3 − Y , factoring out the linear factor and solving the quadratic inequality by quadratic formula gives
2 ≤ Y < 2 5 + 6 2 6 ≈ 5 0 . 0 2 ( 1 )
Now consider a quadratic equation with roots R and G , by Vieta's formula - forming quadratics , we have
X 2 − Y X + 2 0 0 Y 3 − Y = 0
Since Y = R + G is an integer, then the quadratic discriminant of the equation above is a perfect square:
Y 2 − 4 ( 2 0 0 Y 3 − Y ) = M 2 ⇔ Y 3 − Y ≡ 0 ( m o d 5 0 ) ⇔ R + G = Y ≡ 0 , ± 1 ( m o d 2 5 ) ( 2 )
Hence, the possible values of Y = R + G boils down to { 2 4 , 2 5 , 2 6 , 4 9 , 5 0 } .
Now just plug in these 5 possible solutions and with enough patience, we can show that R + G = 4 9 is the only solution.
Hey thanks, Brian! Yeah, proving that this is the solution gets kinda tedious... Oh well, glad you liked it! :)
boiling down to 200r(t-r-1)=t(t-1)(t-2); r=reds; t=total
wolframalpha solveinteger 200r(t-r-1)=t(t-1)(t-2), r>0, t>0
3 solutions red=1; total=2; green=0 contradiction
red=21; t=50; green=28
red=28; t=50; green=21 *)
Problem Loading...
Note Loading...
Set Loading...
Let P ( R , G , B ) = The probability of choosing 3 different colors if there are R reds, G greens, and B blues.
P ( R , G , B ) = ( 3 R + G + B ) R × G × B
For B = 1 , this becomes,
P ( R , G , 1 ) = ( 3 R + G + 1 ) R × G
This only equals . 0 3 if R = 2 1 and G = 2 8 or R = 2 8 and G = 2 1 .
So the total number of balls in the bag in the beginning was 2 1 + 2 8 + 1 = 5 0